Đến nội dung

ilovelife nội dung

Có 362 mục bởi ilovelife (Tìm giới hạn từ 13-05-2020)



Sắp theo                Sắp xếp  

#417890 Đề thi thử lớp 9 vòng 1 đợt 3 trường THPT chuyên KHTN

Đã gửi bởi ilovelife on 11-05-2013 - 21:34 trong Tài liệu - Đề thi

Bôi đỏ để làm gì nhỉ .Ai làm bài hình đi

Bài hình:
Phần $a)$ thì dễ rồi, phần $b)$

Chứng minh $\angle ACB = \angle NHC (=\angle FHA)$ dựa vào $\angle FKA = \angle BOA$

Từ đó suy ra $\triangle NCH \sim \triangle NAC \implies NC^2 = ND^2 = NH.NA$

$\implies NC = ND = \frac 12 CD$

Tương tự $MD = \frac 12 BD$

$\implies MN = \frac 12 BC$ không đổi




#417877 Đề thi thử lớp 9 vòng 1 đợt 3 trường THPT chuyên KHTN

Đã gửi bởi ilovelife on 11-05-2013 - 20:52 trong Tài liệu - Đề thi

Cách phản chứng có vẻ không tự nhiên cho lắm .Mà cái mũ 3 chuyển xuống mũ 2 là ntn :ukliam2:

 

Cái thằng Ngọc Anh này, cứ giả vờ Never Give Up, phân tích thành nhân tử rồi chia 2 vế cho $(a+b-2)$ vì $a+b-2>0$ mà, bây giờ khinh không trả lời đâu




#417872 Đề thi thử lớp 9 vòng 1 đợt 3 trường THPT chuyên KHTN

Đã gửi bởi ilovelife on 11-05-2013 - 20:42 trong Tài liệu - Đề thi

Xin lỗi các bạn!!! Có vẻ như mình bị ngược dấu rồi!!! 

Nhưng có vẻ là $a+b \le 2$ là đúng thật (bạn xem chứng minh của mình ở bên trên có đúng không), mà chứng minh được vậy sẽ suy luôn được $2\sqrt {ab} \le a+b \le 2 \implies ab \le 1$

 

@trananh2771998: giả sử trái lại là giả sử $a+b>2$, mày cứ check bài tao đi, tao làm qua, vội nên có thể sai sót :okay:




#417861 Đề thi thử lớp 9 vòng 1 đợt 3 trường THPT chuyên KHTN

Đã gửi bởi ilovelife on 11-05-2013 - 20:25 trong Tài liệu - Đề thi



Bài $4$:

Ta có: $\left\{\begin{matrix} a^{3}+1+1\geq 3a & \\ b^{3}+1+1\geq 3b & \end{matrix}\right. \Rightarrow a^{3}+b^{3}+6ab+4\leq 12\Rightarrow 3a+3b+6ab\leq 12\Rightarrow a+b+2ab\leq 4$

Lại có: $a+b\geq 2\sqrt{ab}\Rightarrow 4\geq a+b+2ab\geq 2ab+2\sqrt{ab}\Rightarrow ab+\sqrt{ab}-2\leq 0\Rightarrow (\sqrt{ab}-1)(\sqrt{ab}+2)\leq 0\Rightarrow -2\leq \sqrt{ab}\leq 1\Rightarrow ab\leq 1\Rightarrow a+b\leq 2$

Ta có: $\frac{1}{a^{2}+b^{2}}+\frac{1}{2ab}+\frac{1}{ab}+ab+\frac{3}{2ab}\geq \frac{4}{(a+b)^{2}}+\frac{3}{2ab}+2\geq 1+\frac{3}{2}+2=\frac{9}{2}$

Vậy $\max P=\frac{9}{2}\Leftrightarrow a=b=1$

Cho mình hỏi, tại sao, có $ab \le 1$ mà bạn suy ra được $a+b \le 2$ (cùng câu hỏi với "bạn" trananh2771998)

Mà để chứng minh $ab \le 1$ sao bạn không chứng minh $8+1=(a^3+b^3+1)+6ab \ge 9ab \implies 1\ge ab$

 

Chứng minh $a+b\le 2$

Giả sử trái lại, khi ấy:

$(a+b)^3 - 3ab(a+b)+6ab \le 8\\ \implies (a+b)^3-8 \le 3ab (a+b-2)$

$\implies (a+b)^2+2(a+b)+4 \le 3ab \implies$ vô lí

$\implies Q.E.D$

 

Đến đây thì các bạn làm điểm rơi bình thường sẽ tìm được min




#417795 tìm các số tự nhiên a,b,c sao cho $a^{2}(b+c)+b^{2}(...

Đã gửi bởi ilovelife on 11-05-2013 - 16:56 trong Đại số

tìm các số tự nhiên a,b,c sao cho $a^{2}(b+c)+b^{2}(c+a)+c^{2}(a+b)$ là số nguyên tố

Theo mình thì bài này bạn xét các trường hợp dư khi chia cho $3$

Để chứng minh khi ... thì $3 \mid a^{2}(b+c)+b^{2}(c+a)+c^{2}(a+b)$

còn lại thì $2 \mid a^{2}(b+c)+b^{2}(c+a)+c^{2}(a+b)$

$\implies$ chỉ có các nghiệm tầm thường




#417794 Giải phương trình nghiệm nguyên $y^{4}+4y=24$

Đã gửi bởi ilovelife on 11-05-2013 - 16:42 trong Phương trình, hệ phương trình và bất phương trình

Giải phương trình nghiệm nguyên $y^{4}+4y=24$

$24\equiv 4y \equiv 0 \pmod 4\implies 4 \mid y^4 \iff 2|y$

Đặt $y = 2k$ với $k \in \mathbb Z$

$\implies k(k^3+1)=3= \pm 1 \cdot (\pm 3)$

Đến đây thì dễ rồi




#417792 Nhờ các Pro giải giúp út bài toán sau:

Đã gửi bởi ilovelife on 11-05-2013 - 16:16 trong Phương trình - hệ phương trình - bất phương trình

x2+ x1/2 = 5

Hàm trên đồng biến trên tập xác định ($x \in \mathbb{R^+}$)

Do đó phương trình có nghiệm duy nhất
$x= \left( -1/12\,\sqrt {6}\sqrt {{\frac { \left( 108+12\,\sqrt {96081}\right) ^{2/3}-240}{\sqrt [3]{108+12\,\sqrt {96081}}}}}+1/12\,\sqrt {
6}\sqrt {{\frac {-\sqrt {{\frac { \left( 108+12\,\sqrt {96081}\right) ^{2/3}-240}{\sqrt [3]{108+12\,\sqrt {96081}}}}} \left( 108+12\,\sqrt {96081} \right) ^{2/3}+240\,\sqrt {{\frac { \left( 108+12\,\sqrt {96081} \right) ^{2/3}-240}{\sqrt [3]{108+12\,\sqrt {96081}}}}}+12\,\sqrt {6}\sqrt [3]{108+12\,\sqrt{96081}}}{\sqrt [3]{108+12\,\sqrt {96081}}\sqrt {{\frac { \left( 108+12\,\sqrt {96081} \right) ^{2/3}-240}{\sqrt [3]{108+12\,\sqrt {96081}}}}}}}} \right) ^{2}$

hay http://www.wolframal...^2 + sqrt x = 5




#417786 Tìm GTNN P=$\frac{1}{2+a^2}+\frac{1...

Đã gửi bởi ilovelife on 11-05-2013 - 15:44 trong Bất đẳng thức và cực trị

Tìm giá trị nhỏ nhất của P =$\frac{1}{2+a^2}+\frac{1}{2+b^2}+\frac{1}{2+c^2}$

 

với a,b,c dương và a+b+c=3.

$a+b+c=3$ hay là $ab + bc + ca = 3$ (để thế mình mới làm được), bạn xem lại cho mình được không ?




#417580 Bất đẳng thức

Đã gửi bởi ilovelife on 10-05-2013 - 11:27 trong Bất đẳng thức - Cực trị

Ta có đánh giá:

 

$\sum \frac{x}{ y^{2} \dotplus z ^{2}} = \sum \frac x{1-x^2} \ge \sum \frac 32 \sqrt 3 x^2=VP$

 

 

Cho các số dương x,y,z thoả mãn $x^{2} \dotplus y ^{2} \dotplus z^{2} =1$

Chứng minh

$\sum \frac{x}{ y^{2} \dotplus z ^{2}} \geq \frac{3\sqrt{3}}{2 }$




#417454 cực trị với số tự nhiên

Đã gửi bởi ilovelife on 09-05-2013 - 16:17 trong Bất đẳng thức và cực trị

Gợi ý:

 

Theo bài ra, có:
$n \sqrt 6 > m \implies 6n^2 > m^2 \implies 6n^2 \ge m^2+1$

Mà $6 \nmid m^2 + 1\implies 6n^2 \ge m^2 + 2$

$\implies 6 \ge ? > ...$ (bạn tự chứng minh tiếp, chắc hướng làm là như vậy)




#417445 Cực trị hình học liên quan đến diện tích tam giác

Đã gửi bởi ilovelife on 09-05-2013 - 15:22 trong Hình học

Trên các cạnh AB, AC, BC, của tam giác ABC lần lượt lấy $C_{1}, A_{1}, B_{1}$ sao cho $AA_{1}, BB_{1}, CC_{1}\leq 1$. Chứng minh $S_{ABC}\leq \frac{1}{\sqrt{3}}$

Giải như sau:
Không mất tính tổng quát, giả sử $\alpha \ge \beta \ge \gamma$

TH1: $\triangle ABC$ nhọn

$\implies 60^o \le \alpha < 90^o \implies S_{ABC}= \frac 1{2\sin \alpha} h_b\cdot h_c \le \frac 13$

TH2: $\triangle ABC$ không nhọn

$\implies \alpha \ge 90^o \implies S_{ABC}= \frac 12 AB\cdot AC \cdot \sin \alpha \le \frac 12 AB\cdot AC \le \frac 12 < \frac 1{\sqrt3}$




#417294 Topic các bài toán số học dành cho các bạn chuẩn bị thi tuyển sinh 10 năm 201...

Đã gửi bởi ilovelife on 08-05-2013 - 18:26 trong Số học

Tiếp tục nào :D

Bài 73* : Chứng minh rằng phương trình sau không có nghiệm nguyên dương :

a.$x^{4}+y^{4}=z^{2}$

b.$x^{4}-y^{4}=z^{2}$

Bài 74 : Có tồn tại hay không các số nguyên $x$,$y$ thoả mãn điều kiện $1992x^{1993}+1993y^{4}=1995$

Bài 75 : Giải phương trình nghiệm nguyên : $x^{2}+x=y^{4}+y^{3}+y^{2}+y$

Bài 74: Xét đồng dư cho $4 \implies$ vô nghiệm

Bài 75: Nhân thêm $4$ rồi cộng $1$ vào $2$ vế, rồi chặn sẽ tìm đuợc nghiệm

Bài 73: a), b)
Đưa về phương trình pytago, chắc biểu diễn các nghiệm 1 hồi $\implies$ vô nghiệm (chưa thử, không biết lùi vô hạn được không)

EDIT: Đây là 1 bài toán của Nagell, đã được chứng minh vô nghiệm (các bạn thử Google xem có solution không ?).




#417156 Tìm các số nguyên tố $p$ sao cho $\frac{1}...

Đã gửi bởi ilovelife on 07-05-2013 - 21:21 trong Số học

Tìm các số nguyên tố $p$ sao cho $\frac{1}{p}=\frac{1}{a^2}+\frac{1}{b^2}$ với $a,b$ là các số nguyên dương.

 

-------------------------------------------

p/s: Mình được thầy gợi ý là:

Giả sử $(a,b)=m$ suy ra $a=mq,b=mr$ với $(q,r)=1$. Do đó: $p(r^2+q^2)=m^2q^2r^2$

Từ đó lập luận để $q=r=1$

Bài toán mạnh hơn, giải phương trình nghiệm nguyên dương với $p$ là số nguyên tố: $\frac 1p=\frac 1a+\frac 1b$

 





#417153 Chứng minh rằng tồn tại một tam giác có ba đỉnh lấy từ 2015 điểm đã cho có di...

Đã gửi bởi ilovelife on 07-05-2013 - 21:14 trong Hình học

Ta đi chứng minh có $4024$ tam giác không có điểm chung, cách chứng minh khá giống http://diendantoanho...g-nhỏ-rồi-để-t/




#416478 Một số bài tập hình học hay và khó

Đã gửi bởi ilovelife on 04-05-2013 - 20:56 trong Hình học

Phần đặc sắc hơn

Bài 5: Cho hình vẽ, hãy chứng minh $B,L,C$ thẳng hàng

Spoiler

 

Bài 6: Gọi $r$ là bán kính đường tròn nội tiếp ABCD (và ABCDtứ giác nội tiếp). Chứng minh $AB + CD \ge 4r$

Bài 7: Một tam giác có trung điểm các đường cao thẳng hàng. Hỏi tam giác đó là tam giác gì ? (mình nghĩ nó là tam giác vuông)

Bài 8 (bài này khá đơn giản): Trong 5 đường tròn luôn có 4 đường tròn có 1 điểm chung. Chứng minh cả 5 đường tròn đều đi qua điểm chung ấy

 

 




#415810 Tổng MB^2 + MC^2

Đã gửi bởi ilovelife on 01-05-2013 - 15:06 trong Hình học

nếu lấy luôn kết quả định lý cosin mà không cần chứng minh lại thì có thể làm như sau:

trong tam giác ABM có:

 

$AB^{2}+BM^{2}-2.AB.BM.cosB = AM^{2}<=> a^{2}+BM^{2}-2.a.BM.\frac{\sqrt2}{2} = 3^{2} <=> a^{2}+BM^{2}-BM.a.\sqrt2 = 9 <=> a^{2}+BM^{2}-BM.(BM+MC) = 9 <=> BM.MC = a^{2} - 9$

Mà $BM+MC = BC = a.\sqrt2$

=> tính đc BM, MC 

=> tính đc tổng . . . 

À à, em vẽ sai hình nên nó mới lộn linh tinh :D, tưởng nó cân tại $B$




#415802 Tổng MB^2 + MC^2

Đã gửi bởi ilovelife on 01-05-2013 - 14:37 trong Hình học

chỗ dòng đỏ phải là góc giữa vecto AB và AM.

chỗ dòng xanh phải là (vecto AC - vecto AM)^2. do vậy không thể thay độ dài đoạn thẳng AC, AM vào như thế kia vì vecto là 1 đại lượng có hướng.

==>>> nhầm lung tung cả rồi

ughhh.gif

Chị xem lại cái link em gửi đi, em có lôi vector vào đây đâu




#415798 CMR độ dài mỗi cạnh t.giác đều là ước số của chu vi, ít nhất 2 ca...

Đã gửi bởi ilovelife on 01-05-2013 - 14:25 trong Hình học

Cho tứ giác ABCD có độ dài bốn cạnh a, b, c, d đều là các số nguyên dương. CMR nếu độ dài mỗi cạnh đều là ước số của chu vi tứ giác này thì tứ giác đó có ít nhất 2 cạnh bằng nhau.

Theo đầu bài, có:
$\sum a = ma=np=pc=qd \\ \implies \sum \frac 1m = \sum \frac {a}{a+b+c+d} = 1$

Giả sử không có số nào bằng nhau và $a>b>c>d \ge 1 \implies m \ge 3$
$\implies \sum \frac 1m \le 1/3 + 1/4 + 1/5 + 1/6 < 1\implies Q.E.D$




#415795 Tổng MB^2 + MC^2

Đã gửi bởi ilovelife on 01-05-2013 - 14:14 trong Hình học

Ban tham khảo cái: http://en.wikipedia..../Law_of_cosines

Đặt $AB = a$

Áp dụng công thức, có:

 $MB^2 = a^2 + 3^2 - 2a\cdot 3 \cos 45^o$

 $MC^2 = (AC - AM)^2 = (a\sqrt 2 - 3)^2$

Cộng lại...




#415630 Trên bàn có 10 miếng giấy. Người ta chọn một vài miếng và cắt mỗi miếng thành...

Đã gửi bởi ilovelife on 30-04-2013 - 19:48 trong Số học

Trên bàn có 10 miếng giấy. Người ta chọn một vài miếng và cắt mỗi miếng thành 7 miếng nhỏ, rồi để tất cả các miếng giấy sau khi cắt trở lại mặt bàn. Tiếp theo, lại chọn một vài miếng giấy trên bàn (trong tổng số các miếng giấy) và lại cắt mỗi miếng thành 7 miếng rồi để tất cả các miếng giấy sau khi cắt trở lại mặt bàn. Quá trình cứ tiếp diễn như vậy, hỏi sau hữu hạn bước, có thu được 2014 miếng giấy hay không?

Không mất tính tổng quát, có thể giả sử mỗi lần chỉ chọn một miếng giấy, khi ấy, 1 miếng --> 7 miếng

$\implies$ số giấy trên mặt bàn tăng thêm $-1+7 = 6$ (miếng)

Vậy sau $334$ bước sẽ có thêm $6 \cdot 334 = 2004$ miếng $\iff$ số giấy trên bàn là $2004+10 = 2014$

 




#415284 $\dfrac{1}{QM}+\dfrac{1}{QN...

Đã gửi bởi ilovelife on 28-04-2013 - 20:56 trong Hình học phẳng

Cho tam giác ABC nhọn nội tiếp (O,R).Gọi Q là tâm đường tròn Ơ-le của tam giác ,M,N,P lần lượt là giao điểm của (O) với QA,QB,QC.(Q là trung điểm HO với H là trực tâm của tam giác ABC)
$CMR:\dfrac{1}{QM}+\dfrac{1}{QN}+\dfrac{1}{QP} \geq \dfrac{3}{R}$
P/s:Bạn nào có ý tưởng nào cứ nêu ra để tham khảo!

Hình như đầu bài có vấn đề ạ (cái chỗ màu đỏ), em nghĩ là sai vì $(O) \cap QA , QB , QC \equiv A, B, C$

@nhuannhi Là lính mới, bạn đọc bài hướng dẫn LaTeX ấy hoặc dùng nút fx.




#415145 Chứng minh $\overline{IK}$ không đổi

Đã gửi bởi ilovelife on 27-04-2013 - 21:50 trong Hình học

Cho mình hỏi là P cố định ko bạn?

bạn à, P cố định, mình sửa đầu bài rồi đấy




#415140 Chứng minh $\overline{IK}$ không đổi

Đã gửi bởi ilovelife on 27-04-2013 - 21:39 trong Hình học

Cho $P$ nằm trong $(O;R)$. Dây $AB \perp CD \equiv P$. Gọi $K$, $I$ là trung điểm của $OP$, $BC$. 

Chứng minh $\overline {IK}$ không đổi (khi $P, O, R$ cố định)

C5p2jFx.png




#415075 Tìm số dư khi chia [$(2+\sqrt{3})^{2011}$]...

Đã gửi bởi ilovelife on 27-04-2013 - 17:02 trong Số học

Tìm số dư khi chia [$(2+\sqrt{3})^{2011}$] cho $3$ với [$a$] là phần nguyên của $a$

Do $2 \pm \sqrt 3$ là nghiệm $x^2-4 x+1 = 0$
nên nếu $s_ n = x_1^n + x_2^n = (2 + \sqrt 3)^n + (2-\sqrt 3)^n$

thì xét đồng dư cho 3, có: $s_{n+2}=4s_{n+1}+s_n \equiv s_{n+1} + s_n$

$\implies s_{n+3} \equiv s_{n+2}+s_{n+1} \equiv s_n - s_{n+1}$

Tương tự $s_{n+6} \equiv s_{n+3}- s_{n+4} \equiv s_{n+3} - (s_{n+3} + s_{n+2}) = - s_{n+2}$

Đến đây thì dễ rồi. (có thể mình làm hơi bị vòng vèo nhưng hướng là là như vậy)




#414961 Đề thi thử vào lớp 10 THPT chuyên năm học 2013-2014

Đã gửi bởi ilovelife on 26-04-2013 - 20:22 trong Tài liệu - Đề thi

Câu 3

b) Vì $x,y,z \in [1;3]\implies \sum (x-1)(x-3) \le 0$

$\iff \sum (x^2 - 4x + 3) \le 0 \iff \sum x^2 \le 4\sum a -9 \le 20-9=11$

Dấu "=" xảy ra khi $(a;b;c) = (1;1;3)$ và hoán vị $\implies Q.E.D$